LSAT and Law School Admissions Forum

Get expert LSAT preparation and law school admissions advice from PowerScore Test Preparation.

 SherryZ
  • Posts: 124
  • Joined: Oct 06, 2013
|
#12320
HI there, thank you in advance for you help!

Oct 2000 LSAT, Sec 4 LR, Q14:

After re-read the question, I figured out why E is right. But I am still not quite understand why A is wrong. Is it because A is too broad ("MANY IDEAS that work well in theory don't work well in practice")??

Thank you so much!

---Sherry
 Emily Haney-Caron
PowerScore Staff
  • PowerScore Staff
  • Posts: 577
  • Joined: Jan 12, 2012
|
#12336
Hi Sherry,

Yep, you figured it out - A is incorrect because it is much too broad to be very strongly supported by the stimulus. Great job!
 Kdup
  • Posts: 31
  • Joined: Aug 14, 2017
|
#40226
Hi Powerscore,

So, I'm having trouble understanding why D is incorrect. I easily eliminated B, C, and E. I thought that D was correct because the stimulus discusses that analog systems when compared to that of digital systems don't work as well and have practical defects. But, the stimulus does not state that digital systems are necessarily preferable to that of analog systems. That's why I eliminated answer choice E. If the stimulus doesn't implicitly say that digital systems are better, how do we know when to infer that?
 nicholaspavic
PowerScore Staff
  • PowerScore Staff
  • Posts: 271
  • Joined: Jun 12, 2017
|
#40265
Hi kdup,

Good questions and let's start with why Answer Option (D) is incorrect. This is a Must Be True question and so we are limited to the information in the stimulus which we must accept as true. (D) gives us the last clause of "for most practical purposes" and nowhere in the stimulus does the author go that far. In fact, the author identifies only one area of inferiority and that is the area of reproduction. There could be many other practical purposes such as the ability to make transmissions from great differences, not interfering with other signals, etc. All of which may be areas where analog coulf be better than digital.

On the other hand Answer (E) focuses on the reproduction and does pass the Fact Test because most of the stimulus is discussing the practicality of reproduction in analog vs. digital. It is therefore a supportable inference and the correct answer. Thanks for the great question and I hope this helps!

Get the most out of your LSAT Prep Plus subscription.

Analyze and track your performance with our Testing and Analytics Package.